Đến nội dung

Stranger411 nội dung

Có 85 mục bởi Stranger411 (Tìm giới hạn từ 29-04-2020)



Sắp theo                Sắp xếp  

#312379 $\frac{1}{2-\cos A}+\frac{1}{2-\cos B}+\frac{1}...

Đã gửi bởi Stranger411 on 24-04-2012 - 10:28 trong Bất đẳng thức - Cực trị

Bài 1: Cho $\Delta ABC$. Chứng minh:
$$\frac{1}{2-\cos A}+\frac{1}{2-\cos B}+\frac{1}{2-\cos C}\ge 2$$


Bài 2: Cho $\Delta ABC$. Chứng minh:
$$\frac{\left( 1-\sin \frac{A}{2} \right)\left( 1+\cos \frac{A}{2} \right)}{\sin \frac{A}{2}\left( 1+\sin \frac{A}{2} \right)}+\frac{\left( 1-\sin \frac{B}{2} \right)\left( 1+\cos \frac{B}{2} \right)}{\sin \frac{B}{2}\left( 1+\sin \frac{B}{2} \right)}+\frac{\left( 1-\sin \frac{C}{2} \right)\left( 1+\cos \frac{C}{2} \right)}{\sin \frac{C}{2}\left( 1+\sin \frac{C}{2} \right)}\ge 2+\sqrt{3}$$



#312383 CMR: $xy+yz+zx\leq 8$

Đã gửi bởi Stranger411 on 24-04-2012 - 10:36 trong Bất đẳng thức và cực trị

Cho $x,y,z$ la nghiệm của hệ pt:$\left\{\begin{matrix} x^2+xy+y^2=3\\ y^2+yz+z^2=16\end{matrix}\right.$
CMR: $xy+yz+zx\leq 8$

Áp dụng bất đẳng thức Cauchy-Schwarz, ta có:
${{\left( xy+yz+zx \right)}^{2}}$
$=\left[ x\left( y+\frac{z}{2} \right)+z\left( y+\frac{x}{2} \right) \right]$
$\le \left[ {{x}^{2}}+\frac{4}{3}{{\left( y+\frac{x}{2} \right)}^{2}} \right]\left[ {{\left( y+\frac{z}{2} \right)}^{2}}+\frac{3}{4}{{z}^{2}} \right]$
$=\frac{4}{3}\left( {{x}^{2}}+xy+{{y}^{2}} \right)\left( {{y}^{2}}+yz+{{z}^{2}} \right)=64$
$\Leftrightarrow xy+yz+zx\le 8$ (đpcm)



#312448 $\frac{1}{2-\cos A}+\frac{1}{2-\cos B}+\frac{1}...

Đã gửi bởi Stranger411 on 24-04-2012 - 19:17 trong Bất đẳng thức - Cực trị

Hình đã gửiÔng bạn của mình đây mà
Trước hết ta sẽ chứng minh $$cosA+cosB+cosC\geq -\frac{3}{2}$$
Cho $\overrightarrow{OA};\overrightarrow{OB};\overrightarrow{OC}$ là 3 véc tơ đơn vị. ( các góc $A,B,C \in (0; \pi)$)
Ta dễ dàng chứng minh được $(\overrightarrow{OA}+\overrightarrow{OB}+\overrightarrow{OC})^2=3+2(cosA+cosB+cosC) \ge 0$
Từ đây suy ra $$cosA+cosB+cosC\geq \frac{-3}{2}$$
Áp dụng bất đẳng thức AM-HM ta có:
$$\frac{1}{2-cosA}+\frac{1}{2-cosB}+\frac{1}{2-cosC}\geq \frac{9}{3-cosA-cosB-cosC}\geq \frac{9}{3+\frac{3}{2}}=2$$

Một lỗi sai khá cơ bản đó ông bạn Hình đã gửi
$$2\cos (\overrightarrow{OA},\overrightarrow{OB})= \cos2C$$
Từ đó, ta có:
$$cos2A+cos2B+cos2C\geq \frac{-3}{2}$$
Bài này không đơn giản như vậy đâu



#312453 Topic: INEQUALITIES (PART II)

Đã gửi bởi Stranger411 on 24-04-2012 - 19:32 trong Bất đẳng thức - Cực trị

Problem 8: Cho a,b,c là các số thực thỏa $a+b+c=3$. Chứng minh rằng:
$$\frac{a^2-bc}{a^2+3}+\frac{b^2-ac}{b^2+3}+\frac{c^2-ab}{c^2+3}\geq 0$$

Bài này khá yếu.
Bằng cách phân tích trực tiếp, ta được:
$$\left( {{a}^{2}}+{{b}^{2}}+{{c}^{2}}-ab-bc-ca \right)\sum{\frac{{{\left( a-b \right)}^{2}}}{\left( {{a}^{2}}+3 \right)\left( {{b}^{2}}+3 \right)}}\ge 0$$
Ta có đpcm. $\blacksquare$

Problem 9: Cho a,b,c là các số thực thỏa $a+b+c=1$. Chứng minh rằng:
$$\frac{\left( {{a}^{2}}+{{b}^{2}} \right)}{{{\left( a+b \right)}^{2}}}\frac{\left( {{c}^{2}}+{{b}^{2}} \right)}{{{\left( c+b \right)}^{2}}}\frac{\left( {{a}^{2}}+{{c}^{2}} \right)}{{{\left( a+c \right)}^{2}}}\ge \frac{3}{8}\left( {{a}^{2}}+{{b}^{2}}+{{c}^{2}} \right)$$



#312558 Bất đẳng thức $4$ biến thoả $ abcd=1$

Đã gửi bởi Stranger411 on 25-04-2012 - 08:17 trong Bất đẳng thức - Cực trị

Cho các số thực dương $a,b,c,d$ thoả mãn: $abcd=1$.Chứng minh rằng:
$$ 3(a^2+b^2+c^2+d^2) + 4 \ge (a+b+c+d)^2$$

Áp dụng bất đẳng thức Tukervici, ta có:
$ 3(a^4+b^4+c^4+d^4) + 4abcd \ge (a^2+b^2+c^2+d^2)^2$
Bất đẳng thức cần chứng minh là dạng tương đương của bất đẳng thức trên. $\blacksquare$
Bất đẳng thức trên dùng pp FMPX để chứng minh.



#312728 Cho $a,b,c>0$và $ab+bc+ca+abc=4$ .CM$a^3+b^3+c^3...

Đã gửi bởi Stranger411 on 25-04-2012 - 23:19 trong Bất đẳng thức và cực trị

Cho$a,b,c>0$ thỏa mãn $ab+bc+ca+abc=4$.Chứng minh
$a^3+b^3+c^3+9abc\geq 4(a+b+c)$

Phải nói đây một bất đẳng thức khá bá đạo.
Nhưng nếu làm bđt nhiều thì cũng không khó gì để nhận ra nó.
Đổi biến $a=\frac{2x}{y+z},b=\frac{2y}{x+z},c=\frac{2z}{x+y} $, ta được $ab+bc+ca+abc=4$
Lời giải còn lại của bài toán các bạn xem trong file đính kèm.

File gửi kèm




#313126 Bài 1. cho $a,b,c>0$có $abc=1$ chứng minh rằng $...

Đã gửi bởi Stranger411 on 28-04-2012 - 14:02 trong Bất đẳng thức - Cực trị

Bài 1. cho $a,b,c>0$có $abc=1$ chứng minh rằng
$\frac{1}{a^3+b+c}+\frac{1}{b^3+a+c}+\frac{1}{c^3+a+b}\leq \frac{3}{a+b+c}$
Bài 2.cho $a,b,c>0 abc=1$ tìm GTNN của
$P=\frac{1}{2a^3+b^3+c^3+2}+\frac{1}{a^3+2b^3+c^3+2}+\frac{1}{a^3+b^3+2c^3+2}$
Bài 3.chứng minh:
$\frac{a^3}{a^2+ab+2b^2}+\frac{b^3}{b^2+bc+2c^2}+\frac{c^3}{c^2+ab+2a^2}\geq \frac{a+b+c}{4}$
Bài 4. $ x+y+z=3$,tìm giá trị nhỏ nhất của biểu thức

$P=\frac{x}{y^3+16}+\frac{y}{z^3+16}+\frac{z}{x^3+16}$

nếu ko có trả lời một tuần nữa minh sẽ post dáp án

Có lẽ cũng chẳng cần post đáp án nữa đâu bạn :lol:
Bài 2:
Áp dụng bđt Cauchy, ta đc:
$$ \frac{1}{2{{a}^{3}}+{{b}^{3}}+{{c}^{3}}+2}\le \frac{1}{4}\left( \frac{1}{{{a}^{3}}+{{b}^{3}}+1}+\frac{1}{{{a}^{3}}+{{c}^{3}}+1} \right) $$
Từ đó, ta được: $ P\le \frac{1}{2}\left( \frac{1}{{{a}^{3}}+{{b}^{3}}+1}+\frac{1}{{{a}^{3}}+{{c}^{3}}+1}+\frac{1}{{{c}^{3}}+{{b}^{3}}+1} \right)\le \frac{1}{2} $

Vì $ {{a}^{3}}+{{b}^{3}}+1\ge ab\left( a+b \right)+1=ab\left( a+b+c \right) $
Bài 3: Sử dụng pp tiếp tuyến, ta chứng minh:

$$ \frac{{{a}^{3}}}{{{a}^{2}}+ab+2{{b}^{2}}}\ge \frac{9a-5b}{16} $$
Cộng các bđt tương tự, ta có đpcm.



#313291 ${{a}^{3}}+{{b}^{3}}+{{c}^{3}}+kabc\ge 3+k$

Đã gửi bởi Stranger411 on 29-04-2012 - 14:32 trong Bất đẳng thức - Cực trị

Mình đang viết 1 chuyên đề về phương pháp tìm hằng số lớn nhất cho bđt.
Mong các bạn góp ý :D

Bài toán 1: Cho các số thực không âm a,b,c thỏa mãn $a+b+c=3$. Tìm hằng số k lớn nhất để bđt sau đúng:

$${{a}^{3}}+{{b}^{3}}+{{c}^{3}}+kabc\ge 3+k$$

Bài toán 2: (Stranger411)Cho các số thực không âm a,b,c thỏa $ab+bc+ca=3$. Tìm hằng số k lớn nhất để bđt sau đúng:

$${{a}^{3}}+{{b}^{3}}+{{c}^{3}}+kabc\ge 3+k$$

Chú ý: Với bài toán 2,ta có một số kết quả:

$k=6$: Hệ quả của bđt Schur
$k=7$: Bài toán đã được arqady giải ở Mathlinks
Tuy nhiên,$k=7$ không phải hằng số tốt nhất cho bđt trên.



#313293 Balkan MO 2012

Đã gửi bởi Stranger411 on 29-04-2012 - 14:46 trong Thi HSG Quốc gia và Quốc tế

Balkan MO 2012 - 28 April 2012

Bài 2. Prove that \[ \sum_{cyc}(x+y)\sqrt{(z+x)(z+y)}\geq 4(xy+yz+zx), \] for all positive real numbers $x,y$ and $z$.

Tuy là đề thi quốc gia nhưng mình thấy bài này khá lỏng :D
Cách 1:
Áp dụng bđt Cauchy-Schwarz, ta có:


$\left( x+y \right)\sqrt{\left( z+x \right)\left( z+y \right)}\ge \left( x+y \right)\left( z+\sqrt{xy} \right)$

$=\left( x+y \right)z+\left( x+y \right)\sqrt{xy}\ge \left( x+y \right)z+2xy$
Cách 2: Bằng cách dùng bổ đề của huymit_95, ta viết lại bđt như sau:
$\sum {\frac{1}{{\sqrt {\left( {x + y} \right)\left( {x + z} \right)} }}} \ge \frac{{4\left( {xy + yz + zx} \right)}}{{\left( {x + y} \right)\left( {y + z} \right)\left( {z + x} \right)}}$
Ta có: $\left( {x + y} \right)\left( {y + z} \right)\left( {z + x} \right) \ge \frac{8}{9}\left( {xy + yz + zx} \right)\left( {x + y + z} \right)$
Nên ta cần chứng minh: $\sum {\frac{1}{{\sqrt {\left( {x + y} \right)\left( {x + z} \right)} }}} \ge \frac{9}{{2\left( {x + y + z} \right)}}$
Và bđt này hoàn toàn đúng với bđt Cauchy-Schwarz:
$\sum {\frac{1}{{\sqrt {\left( {x + y} \right)\left( {x + z} \right)} }}} \ge \sum {\frac{2}{{x + y + 2z}}} \ge \frac{9}{{2\left( {x + y + z} \right)}}$



#313754 $$\dfrac{b(a+b)}{(c+a)^2}+\dfrac{c(c+b)}{(a+b)^2}+\d...

Đã gửi bởi Stranger411 on 01-05-2012 - 20:13 trong Bất đẳng thức và cực trị

Bài toán 3.
Cho các số thực dương $a, b, c$ sao cho $abc=1$. Chứng minh rằng :
$$\dfrac{1}{(1+a)^2(b+c)}+\dfrac{1}{(1+b)^2(c+a)}+\dfrac{1}{(1+c)^2(a+b)}\le \dfrac{3}{8}$$


Trần Quốc Anh


Hình đã gửi
Trước tiên, ta chứng minh bổ đề:
$$(a+1)(b+c)\ge \frac{(b+1)(c+1)}{\sqrt{bc}}$$
Có thể dùng Cauchy-Schwwarz để chứng minh hoặc biến đổi tương đương ;)

Bất đẳng thức cần chứng minh trở thành:
$$\sum{\frac{1}{{{(a+1)}^{2}}(b+c)}}\le \sum{\frac{bc}{(a+1)(b+1)(zc+1)}}$$

Vậy, ta chỉ cần chứng minh:
$$(a+1)(b+1)(c+1)\ge \frac{8}{3}(\sqrt{ab}+\sqrt{bc}+\sqrt{ca})$$
Và đây là hệ quả của bđt:

$$(x+y)(y+z)(z+x)\ge \frac{8}{9}(x+y+z)(xy+yz+zx)$$
Bất đẳng thức đã được chứng minh. $\blacksquare$


Ps: anh phuc_90 vào góp vui đi ak :P
Bài 1: Giải bằng SOS mất 3 trang giấy :-ss Mọi người ai có cách giải ngắn hơn ko :-ss



#314274 $$\dfrac{a^3+b^3+c^3+3abc}{(a+b+c)(ab+bc+ca)}+\dfrac{abc}...

Đã gửi bởi Stranger411 on 04-05-2012 - 13:03 trong Bất đẳng thức - Cực trị

Bài toán :
Cho $a, b, c \ge 0$ . Chứng minh rằng :
$$\dfrac{a^3+b^3+c^3+3abc}{(a+b+c)(ab+bc+ca)}+\dfrac{abc}{a^2b+b^2c+c^2a}\ge 1$$
Nguồn : ML

Bài toán của bạn được ghép từ 3 bổ đề sau:
1)${{a}^{3}}+{{b}^{3}}+{{c}^{3}}+3abc\ge \sum{ab\left( a+b \right)}$


2)$\left( a+b+c \right)\left( ab+bc+ca \right)\frac{8}{9}\le \left( a+b \right)\left( b+c \right)\left( c+a \right)$

3)$\frac{a}{b+c}+\frac{b}{c+a}+\frac{c}{a+b}+\frac{3abc}{2\left( {{a}^{2}}b+{{b}^{2}}c+{{c}^{2}}a \right)}\ge 2$



#339875 $\sum\limits_{k=1}^{p-1}{({...

Đã gửi bởi Stranger411 on 25-07-2012 - 07:42 trong Tổ hợp và rời rạc

Cho số nguyên tố $p>3$ và tập hợp $M=\left\{ 1,2,...,p \right\}$. Với mỗi số nguyên $k$ thỏa mãn $1\le k\le p$ ta đặt : ${{E}_{k}}=\left\{ A\subset M:|A|=k \right\}$ và ${{x}_{k}}=\sum\limits_{A\in {{E}_{k}}}{\left( \min A+\max A \right)}$. Chứng minh rằng:
$$\sum\limits_{k=1}^{p-1}{({{x}_{k}}C_{p}^{k})\equiv0(\bmod \,\,{{p}^{3}})}$$



#339925 $\sum\limits_{k=1}^{p-1}{({...

Đã gửi bởi Stranger411 on 25-07-2012 - 09:53 trong Tổ hợp và rời rạc

Có vẻ như bài này chỉ chứng minh chia hết cho $p^2$ thôi, thử với $p=3$ có vẻ không đúng.

Rõ ràng là :
$$ \sum\limits_{k=1}^{p-1}(C_p^{k})^2 \vdots p^2$$

Anh gì đó ơi, bài này có thể quy về chứng minh:
\[\sum\limits_{k=1}^{p-1}{{{\left( C_{p}^{k} \right)}^{2}}\equiv 1(\bmod \,\,{{p}^{3}})}\]
$$\Leftrightarrow \sum\limits_{k=1}^{p-1}{{{\left( \frac{(p-1)!}{k!(p-k)!} \right)}^{2}}\equiv 0(\bmod \,\,\,p)}$$


Với mỗi $k\in \text{ }\!\!\{\!\!\text{ 1}\text{,2},...,\text{ p-1}\}$ đặt ${{a}_{k}}=\frac{(p-1)!}{k!(p-k)!}$
$$ \Leftrightarrow k!.{{a}_{k}}=(p-1)(p-2)...(p-k+1) $$
$$ \Leftrightarrow k.{{a}_{k}}\equiv {{(-1)}^{k-1}}(\bmod \,\,\,p) $$

Đến đây, dùng định lí Willson thôi anh ạ :lol:
Mà $p>3$ mà anh :icon6:



#339956 $\sum\limits_{k=1}^{p-1}{({...

Đã gửi bởi Stranger411 on 25-07-2012 - 10:29 trong Tổ hợp và rời rạc

Ở đầu bài toán có đk là p>3
mình sẽ tiếp tục lời giải của bạn để chứng minh chia hết cho $p^3$
ta có $\sum_{k=1}^{p-1}\binom{p}{k}^2=\binom{2p}{p}-2$
mà theo định lí Wolstenholme ta có $\binom{2p}{p} \equiv 2 (mod p^3)$
phát biểu Định lí http://chuyentoanpbc...2/06/trang1.jpg

Em năm nay 12 mà chả biết mấy cái này :mellow:
Em ko bit đánh giá thế nào nên phải dựa vào cách chứng minh của định lí Willson nên nó hơi dài 1 tí :mellow:

Ta chứng minh:
$\sum\limits_{k=1}^{p-1}{{{\left( C_{p}^{k} \right)}^{2}}\equiv 1(\bmod \,\,{{p}^{3}})}$ (1)

$\Leftrightarrow \sum\limits_{k=1}^{p-1}{{{\left( \frac{(p-1)!}{k!(p-k)!} \right)}^{2}}\equiv 0(\bmod \,\,\,p)}$ (2)

Với mỗi $k\in \text{ }\!\!\{\!\!\text{ 1}\text{,2},...,\text{ p-1}\}$ đặt ${{a}_{k}}=\frac{(p-1)!}{k!(p-k)!}$
$ \Leftrightarrow k!.{{a}_{k}}=(p-1)(p-2)...(p-k+1) $
$ \Leftrightarrow k.{{a}_{k}}\equiv {{(-1)}^{k-1}}(\bmod \,\,\,p) $ (3)

Xét ${{b}_{k}}=\frac{(p-1)!}{k}$, $\forall k\in \left\{ 1,2,...,p-1 \right\}$.

Theo Định lý Wison ta có $k{{b}_{k}}\equiv (-1)(\bmod \,\,\,p)$. (4)

Từ (3) và (4) ta có :
${{a}_{k}}\equiv {{(-1)}^{k}}{{b}_{k}}(\bmod \,\,p)$ (5)

Do $p$ là số nguyên tố và $k\in \left\{ 1,2,...,p-1 \right\}$ nên tồn tại duy nhất $j\in \left\{ 1,2,...,p-1 \right\}$ sao cho:
$(kj)\equiv 1(\bmod \,\,\,p)$$\Rightarrow $${{(kj)}^{2}}\equiv 1(\bmod \,\,\,p)$.

Khi đó:
$$\sum\limits_{k=1}^{p-1}{{{({{b}_{k}})}^{2}}}=\sum\limits_{k=1}^{p-1}{\left( {{({{b}_{k}})}^{2}}.1 \right)}\equiv \sum\limits_{k=1}^{p-1}{\left( {{({{b}_{k}})}^{2}}.{{(kj)}^{2}} \right)}\equiv \left( (p-1)! \right)\sum\limits_{j=1}^{p-1}{{{j}^{2}}(\bmod \,\,\,p)}$$

$$\sum\limits_{j=1}^{p-1}{{{j}^{2}}=\frac{p(p-1)(2p-1)}{6}\equiv 0(\bmod \,\,\,p)}$$
nên $\sum\limits_{k=1}^{p-1}{{{({{b}_{k}})}^{2}}}\equiv 0(\bmod \,\,\,p)$ (6)

Từ (5) và (6) suy ra $\sum\limits_{k=1}^{p-1}{{{({{a}_{k}})}^{2}}}\equiv 0(\bmod \,\,\,p)$ hay (2) đúng.



#340308 $S\left( {{a_n}} \right)$ không chia...

Đã gửi bởi Stranger411 on 26-07-2012 - 00:51 trong Tổ hợp và rời rạc

Cho 1 dãy số nguyên $\left( {{a_n}} \right)$ phân biệt thỏa mãn ${{a_n} \leqslant 4999n}$ ${\forall n \geqslant 1}$.
Chứng minh có vô hạn số $n$ sao cho $S\left( {{a_n}} \right) $ không chia hết cho $5$



#340310 Chứng minh: $a = {10^k}$

Đã gửi bởi Stranger411 on 26-07-2012 - 01:01 trong Số học

Cho số nguyên dương $a$ thỏa mãn $S\left( {{a^n} + n} \right) = 1 + S\left( n \right)$ với mọi số tự nhiên $n$ lớn tùy ý.
Chứng minh $a$ là một lũy thừa của $10$.

- Kvant -




#340870 Chứng minh: $a = {10^k}$

Đã gửi bởi Stranger411 on 27-07-2012 - 20:43 trong Số học

Cho n=1.Ta có S(a+1)=2
Suy ra a+1 có dạng:\[a + 1 = 2 \times {10^k}\] hoặc \[a + 1 = {10^k} + {10^h}(k > h)\]

Đến đây có thể dùng 2 tính chất quan trọng của $S(n)$ để giải bài toán.
là $S(m)+S(n) \ge S(m+n)$ và $S(m)S(n) \ge S(mn)$



#343028 có tất cả bao nhiêu số tự nhiên n<m sao cho m l n(2n+1)(5n+2)

Đã gửi bởi Stranger411 on 03-08-2012 - 11:27 trong Số học

cho m=20072008 ,hỏi có tất cả bao nhiêu số tự nhiên n<m sao cho m l n(2n+1)(5n+2)

\[{\text{m}} = {\text{2}}{007^{2008}} = {3^{4016}}{223^{2008}}\]
Ta có: $m|n\left( {2n + 1} \right)\left( {5n + 2} \right)$
$ \Rightarrow m|10n\left( {10n + 5} \right)\left( {10n + 4} \right)$ $(1)$

Đặt $10n=x$, ta được: $m|x\left( {x + 5} \right)\left( {x + 4} \right)$

Đặt ${q_1} = {3^{4016}},{q_2} = {223^{2008}}$. Vì $\gcd \left( {{q_1},{q_2}} \right) = 1$

Nên \[{\text{(1)}} \Leftrightarrow \left\{ \begin{gathered}
x\left( {x + 5} \right)\left( {x + 4} \right) \equiv 0(\bmod {q_1}) \\
x\left( {x + 5} \right)\left( {x + 4} \right) \equiv 0(\bmod {q_2}) \\
\end{gathered} \right.\]
Mà $x \equiv 0(\bmod 10)$

Nên $x$ là nghiệm của hệ:
\[\left\{ \begin{gathered}
x \equiv {r_1}(\bmod {q_1}) \\
x \equiv {r_2}(\bmod {q_2}) \\
x \equiv 0(\bmod 10) \\
\end{gathered} \right.\]
với ${r_1},{r_2} \in \left\{ {0; - 4; - 5} \right\}$

Theo định lí Thặng Dư Trung Hoa, hệ có 1 nghiệm $(\bmod 10{q_1}{q_2})$
với mỗi cặp ${r_1},{r_2}$ chỉ tồn tại 1 nghiệm x.

Có tất cả $3^2$ cách chọn ${r_1},{r_2}$ nên có 9 số $x$ thỏa.
Suy ra có 9 số $n$ thỏa mãn bài toán.



#343066 Chứng minh giá trị của $k$ thuộc 1 tập xác định

Đã gửi bởi Stranger411 on 03-08-2012 - 12:48 trong Số học

Chứng minh rằng nếu tồn tại $ a;b\in\mathbb{Z}^{+} $ sao cho $ \frac{(a-b)^{2}+m}{pab-q}= k (m\in\mathbb{N}^{*}) $ với $p,q$ được xác định như trên thì giá trị của $k$ thuộc 1 tập xác định.



#343069 bài toán về tập tốt

Đã gửi bởi Stranger411 on 03-08-2012 - 12:53 trong Tổ hợp và rời rạc

Bài toán đã có ở đây :D

Bulgaria TST 2003



#343075 MOSP 2001 by Cecil Rousseseau

Đã gửi bởi Stranger411 on 03-08-2012 - 13:15 trong Tổ hợp và rời rạc

Problem: $a_n$ kí hiệu là số tập con không rỗng của $S$ thỏa mãn rằng:
(i) $S\subseteq${$1$, $2$, $...$, $n$};
(ii) tất cả các phần tử của $S$ đều cung tính chẵn, lẻ.
(iii) mỗi phân tử $k\in{S}$ thỏa mãn $k\geq2|S|$.
Tìm công thức tường minh cho $a_n$

Bài này lâu rồi, sử dụng phép chia nhóm là được :)

Ta có: $ a_{2m-1}= 2(F_{m+1}-1) $ và $ a_{2m}= F_{m+3}-2 $
với $m\ge1$ và $F_{m}$ là số Fibonacci thứ $m$.


Lời giải:
Đặt $T_{n}=\{S\in\{1,2,\cdots,n\}\}$ thỏa $(ii)$ và $(iii)$
Chia $ T_{n+4} $ thành 3 tập con:

Phần 1: $A_{n+4}=\{S\in T_{n+4}\ ;\ 1,2\notin S,\ \forall k\in S, k\geq 2|S|+2\}$
Xây dựng $ f\ :\ \mathcal{P}(\{1,2,\cdots,n+2\})\rightarrow\mathcal{P}(\{1,2,\cdots,n+4\}) $ thỏa:
$$f(\{x_{1},x_{2},\cdots,x_{k}\}) =\{x_{1}+2,x_{2}+2,\cdots,x_{k}+2\}$$
Ta được: $ f(T_{n+2}) = A_{n+4} $ nên $ |A_{n+4}|=|T_{n+2}|$

Phần 2: $ B_{n+4}=\{S\in T_{n+4}\ ;\ 1,2\notin S,\ \exists k\in S, k < 2|S|+2\} $
Tương tự, ta được:

$f(\phi) =\{3\}$
$f\left( {\left\{ {{x_1},{x_2},...,{x_k}} \right\}} \right) = \left\{ {{x_1} + 4,{x_2} + 4,...,{x_k} + 4,2k} \right\}$
nếu các phần tử cùng chẳn.
$f\left( {\left\{ {{x_1},{x_2},...,{x_k}} \right\}} \right) = \left\{ {{x_1} + 4,{x_2} + 4,...,{x_k} + 4,2k+1} \right\}$
nếu các phần tử cùng lẽ.
Suy ra: $|B_{n+4}|=|T_{n}|+1 $

Phần 3: $ C_{n+4}=\{ S\in T_{n+4}\ ;\ 1\in S\ \mathrm{or}\ 2\in S\} $
Tương tự, ta có: $ |T_{n+4}|=|T_{n+2}|+|T_{n}|+2 $

Từ đó, ta chứng minh được: $ a_{2m-1}= 2(F_{m+1}-1) $ và $ a_{2m}= F_{m+3}-2 $



#343333 Đếm số cách lát các quân đôminô

Đã gửi bởi Stranger411 on 04-08-2012 - 13:57 trong Tổ hợp và rời rạc

Cho một hình chữ nhật có kích thước $2 \times n$ được đánh số thứ tự từ trái sang phải là $1,2,, \cdot,n$ ở hàng thứ nhất và $n+1,n+2, \cdot,2n$ ở hàng thứ 2.

Lát chúng bằng các quân đôminô $1\times2$ sao cho
1) Phủ kín hình chữ nhật và ko có 2 quân nào chồng lền nhau.
2) Vơi $n$ lẻ, ta được phép bổ sung thêm 1 quân đôminô "đặc biệt" sao cho có thể phủ kín ô $n$ và $n+1$
Đếm số cách lát thỏa mãn đề bài.



ps: Thực chất đây là bài toán tập hợp liên quan đến các phần tử tốt.



#343961 $A = \frac{{{a^2} + {b^2} + {c^2...

Đã gửi bởi Stranger411 on 06-08-2012 - 12:19 trong Số học

Tìm các số nguyên dương a,b,c đề $A = \frac{{{a^2} + {b^2} + {c^2}}}{{abc}} \in {N^*}$

Bài này vô số nghiệm.

Bước 1: Ta cần chứng minh: $A=1$ hoặc $A=3$.

Bước 2: Ta tìm $a,b,c$
Với $A=3$, nghiệm của phương trình $a^2+b^2+c^2=3abc$ là
\[\left\{ \begin{gathered}
z = 1 \\
x = {u_n} \\
y = {u_{n + 1}} \\
\end{gathered} \right.\]
Với
\[\left\{ \begin{gathered}
{u_0} = 1,{u_1} = 1\\
{u_{n + 1}} = 3{u_n} - {u_{n - 1}}\\
\end{gathered} \right.\]

Với $A=1$. Đặt $a=3x,b=3y,c=3z$, ta được phương trình trên.
Dẫn đến một bộ nghiệm tương tự ;)



#344948 CMR$\left( {\frac{{p - 1}}{2...

Đã gửi bởi Stranger411 on 08-08-2012 - 23:38 trong Số học

Bài toán trên còn 1 cách phát biểu khác như sau:
Hình đã gửi



#345818 $\frac{{{a^2} - 2}}{{2...

Đã gửi bởi Stranger411 on 11-08-2012 - 12:21 trong Số học

Tìm $a,b \in {\mathbb{Z}^ + }$ sao cho $\frac{{{a^2} - 2}}{{2{b^2} + 3}} \in \mathbb{Z}$